Page 1 of 1

#12 - Local, Must Be True, Maximum

Posted: Thu Jun 17, 2021 2:42 pm
by Dave Killoran
Complete Question Explanation
(The complete setup for this game can be found here: lsat/viewtopic.php?f=165&t=7597)

The correct answer choice is (B).

The question stem specifies that Sukanya receives two messages from L. This means that either five or six messages must have been sent. The question stem also requests you to identify the greatest number of messages between L’s first message and L’s second message. Given that the idea is to maximize the number of messages between L’s two messages, the template containing six messages is more likely to yield the answer than the template containing five messages because there are more messages to work with.

Both Template 3A and 3B allow for a maximum of one message to be sent between L’s two messages:

G2-Q12-d1.png

Hence, the answer must be at least one, and answer choice (A) can be eliminated.

The templates containing five messages at best allow for no messages between L’s two messages:

G2-Q12-d2.png

Accordingly, the maximum is one, and answer choice (B) is correct.